Re: [obm-l] Problema

2015-07-14 Por tôpico Benedito Tadeu V. Freire
Rogério,

Olá. Muito obrigado.
Benedito

-- 
Open WebMail Project (http://openwebmail.org)

-- Original Message ---
From: Rogerio Ponce abrlw...@gmail.com 
To: obm-l@mat.puc-rio.br obm-l@mat.puc-rio.br 
Sent: Tue, 7 Jul 2015 19:43:31 -0300 
Subject: Re: [obm-l] Problema

 Ola' Benedito,
 Em modulo 5, existem cinco zeros, cinco grupos com 1,2,3,4 (onde 1 e' 
 complemento de 4, e 2 e' complemento de 3) , e o grupo 1,2.
 
 O jogador A vence se chegar ao final com um par complementar (em modulo 5), 
 e mais um numero qualquer, pois basta que ele entao apague este numero.
 
 Assim, o jogador A comeca apagando o 1, por exemplo.
 Imagine que este 1 pertencia ao grupo 1,2.
 Ficou sobrando um 2, que pode ser associado a um dos zeros, formando um par 
 que vou chamar de par estranho.
 
 Agora, alem desse par estranho 0,2 , existem doze grupos de pares 
 complementares ( dos tipos 0,0 , 1,4 e 2,3 ).
 A partir de entao, a cada jogada de B, A apaga o complemento.
 
 Observe que quando B apagar o primeiro dos cinco 0 existentes, A 
 considera que este zero pertence ao par estranho, e apaga o 2 associado.
 Da mesma forma, se B apagar o primeiro dos seis 2 existentes, A 
 considera que este 2 pertence ao par estranho, e entao apaga o 0 
 associado.
 
 Ou seja, sempre que B apagar um numero, A apaga o complemento.
 Ao final, sempre sobrara' um par complementar.
 
 []'s
 Rogerio Ponce
 
 2015-07-06 14:39 GMT-03:00 benedito freire bened...@ufrnet.br:
 
 
 Qual é realmente a estratégia para vencer?
 
---
De: Mauricio de Araujo
 Enviada em: ‎01/‎07/‎2015 14:24
 Para: obm-l@mat.puc-rio.br
 Assunto: Re: [obm-l] Problema
 
 [UTF-8?]​ou melhor, A deve evitar enquanto puder apagar algum múltiplo de 
 [UTF-8?]5.​
 
 Em 1 de julho de 2015 14:21, Mauricio de Araujo 
 mauricio.de.ara...@gmail.com escreveu:
 
 
 A não deve apagar nenhum múltiplo de 5.
 
 Em 1 de julho de 2015 14:19, Mauricio de Araujo 
 mauricio.de.ara...@gmail.com escreveu:
 
 
 [UTF-8?]​Ao final do jogo, A terá apagado 13 números e B 12 números (para 
 que sobre 2 números)... a estratégia vencedora de B seria apagar todos os 
 números 3(mod5) e 4(mod5) além de 3 números 0(mod5) dos quatro existentes, ou 
 seja, teria de executar 13 ações de apagar... como ele só joga 12 vezes A 
 vence sempre (desde que jogue com [UTF-8?]cuidado)..​
 
 Em 1 de julho de 2015 13:30, Pedro José petroc...@gmail.com escreveu:
 
 
 Bom dia !
 Está errado o jogador pode escolher a sobra de E ou F antes de cabarem todos 
 os números. Necessita de reanálise.
 
 -- Mensagem encaminhada --
 De: Pedro José petroc...@gmail.com
 Data: 1 de julho de 2015 10:54
 Assunto: Re: [obm-l] Problema
 Para: obm-l@mat.puc-rio.br
 
 Bom dia!
  
  
 E={1,6,11,16,21,26} e F= {4,9,14,19,24} Para qualquer par (a,b) com a ÆÂ E e 
 b Ɛ F == a + b ≡ 0 (mod5).
 G= {2, 7, 12, 17, 22,27} e H = {3, 8, 13, 18, 23} Para qualquer  (a,b) com a 
 Ɛ G e bƐ H == a + b ≡ 0 (mod5).
 J= {5, 15, 20, 25} Para qualquer par (a,b) com a,b Ɛ J== a + b ≡ 0 (mod5).
  
 O jogador A só ganha se restarem dois números pertencentes a J, um a G e 
 outro a H, um a E e outro a F.
 Portanto o jogador B vence fácil.
  
 Basta para cada escolha  a do jogador A que inicia, o jogador B deve escolher 
 -a | a + (-a) ≡0 (mod5).
  
 Se A escolhe em E, B escolhe em F e vice-versa.
 Se A escolhe em G, B escolhe em H e vice-versa.
 Se A escolhem J, B escolhe em J.
  
 Como a cardinalidade de E e G é maior que a cardinalidade de F e H e a 
 cardinalidade de J é par, ao final sobrarão um elemento s Æ E e t Ɛ  F | s 
 + t ≡ 3 (mod5) 
 
 Saudações,
 PJMS
  
 
 Em 1 de julho de 2015 06:46, bened...@ufrnet.br escreveu:
 
 
 Problema
 
 Dois jogadores, A e B, disputam um jogo, em que jogam alternadamente. O 
 jogador A começa. Uma jogada consiste em apagar um dos números inteiros do 
 conjunto {1, 2, 3,..., 27} até que reste somente dois números. Se a soma 
 desses dois últimos números for divisível por 5, o jogador A vence, caso 
 contrário, vence o jogador B.
 Se cada jogador faz suas melhores jogadas, quem vence: A ou B? Qual é a 
 estratégia para vencer?
 
 --
 Esta mensagem foi verificada pelo sistema de antivírus e 
 acredita-se estar livre de perigo.
 
 --
 Esta mensagem foi verificada pelo sistema de antivírus e 
 acredita-se estar livre de perigo.
 
 
 -- 
 
 Abraços
 
 [UTF-8?]oɾnɐɹɐ [UTF-8?]ǝp [UTF-8?]oıɔıɹnɐɯ
 
 
 
 
 -- 
 
 Abraços
 
 [UTF-8?]oɾnɐɹɐ [UTF-8?]ǝp [UTF-8?]oıɔıɹnɐɯ
 
 
 
 
 -- 
 
 Abraços
 
 [UTF-8?]oɾnɐɹɐ [UTF-8?]ǝp [UTF-8?]oıɔıɹnɐɯ
 
 
 
 [A mensagem original inteira não está incluída.]
 --
 Esta mensagem foi verificada pelo sistema de antivírus e 
 acredita-se estar livre de perigo.
 
 --
 Esta mensagem foi verificada pelo sistema de [UTF-8?]antivírus e 
 acredita-se estar livre de perigo.
--- End of Original Message ---
 

-- 
Esta mensagem foi verificada pelo 

Re: [obm-l] Equação diofantina (de novo)

2015-04-22 Por tôpico Benedito Tadeu V. Freire
Pedro,

7 é o inverso de 7 módulo 12

-- 
Open WebMail Project (http://openwebmail.org)

-- Original Message ---
From: Pedro Chaves brped...@hotmail.com 
To: obm-l@mat.puc-rio.br obm-l@mat.puc-rio.br 
Sent: Wed, 22 Apr 2015 12:46:28 +0300 
Subject: [obm-l] Equação diofantina (de novo)

 Caros Colegas, 
 
 A equação diofantina 7x - 12y = 11 pode ser resolvida por congruência?  Não 
 consegui. 
 
 Sei que 7x é congruente a -1 (mod 12), mas não sei como ir em frente. 
 
 Abraços. 
 Pedro Chaves          
 -- 
 Esta mensagem foi verificada pelo sistema de antivírus e 
 acredita-se estar livre de perigo. 
 
 = 
 Instruções para entrar na lista, sair da lista e usar a lista em 
 http://www.mat.puc-rio.br/~obmlistas/obm-l.html 
 = 
--- End of Original Message ---
 

-- 
Esta mensagem foi verificada pelo sistema de antivírus e
 acredita-se estar livre de perigo.



Re: [obm-l] Problema do Cavalo

2014-02-25 Por tôpico Benedito Tadeu V. Freire
No primeiro passo, existem 8 possibilidades para o cavalo atingir.

-- 
Open WebMail Project (http://openwebmail.org)

-- Original Message ---
From: terence thirteen peterdirich...@gmail.com 
To: obm-l@mat.puc-rio.br 
Sent: Mon, 24 Feb 2014 13:12:27 -0300 
Subject: Re: [obm-l] Problema do Cavalo

 Uma pergunta além: você quer saber quantas casas foram atingidas ao 
 final do percurso, certo? No seguinte sentido: 
 
 No primeiro passo, ele pode atingir até 4 casas. Na segunda, estas 4 
 casas não contam mais, mas apenas os lugares a partir do qual elas 
 chegam. 
 
 Em 19/02/14, Beneditobened...@ufrnet.br escreveu: 
  OK Bernado. 
  Vou dar uma olhada. 
  Obrigado. 
  Benedito 
  
  -Mensagem original- 
  De: owner-ob...@mat.puc-rio.br [mailto:owner-ob...@mat.puc-rio.br] Em nome 
  de Bernardo Freitas Paulo da Costa 
  Enviada em: terça-feira, 18 de fevereiro de 2014 18:00 
  Para: Lista de E-mails da OBM 
  Assunto: Re: [obm-l] Problema do Cavalo 
  
  2014-02-18 14:30 GMT-03:00 Benedito bened...@ufrnet.br: 
  
  É infinito nos quatro quadrantes, que é para permitir muitos movimentos. 
  
  De: owner-ob...@mat.puc-rio.br [mailto:owner-ob...@mat.puc-rio.br] Em 
  nome de terence thirteen Enviada em: segunda-feira, 17 de fevereiro de 
  2014 08:16 
  Para: obm-l 
  Assunto: Re: [obm-l] Problema do Cavalo 
  
  Ele é infinito nos quatro quadrantes? 
  
  Eu tentaria algo como construir um grafo infinito, mas vou pensar 
  antes... 
  
  Eu tenho uma idéia de solução no braço. Supondo que a questão seja: 
  Qual é o número de casas diferentes em que um cavalo pode terminar uma 
  seqüência de N movimentos. Assim, para n = 1, temos 8 casas (brancas), e 
  para n = 2 temos 33 casas (pretas, incluindo a casa preta original!). 
  
  Para n maior, a seqüência fica assim (feito num computador, na marra): 
  
  8; 33; 76; 129; 196; 277; 372; 481; 604; 741; 892; 1057; 1236; 1429; ... 
  
  Agora, vem o chute principal (que é o que vai ajudar a gente a fazer 
  indução): Calcule as diferenças sucessivas dos elementos! Isso dá: 
  
  25; 43; 53; 67; 81; 95; 109; 123; 137; 151; 165; 179; 193; ... 
  
  Ainda não parece bom ? Não tem problema... Mais uma vez, faça as 
  diferenças: 
  
  18; 10; 14; 14; 14; 14; 14; 14; 14; 14; 14; 14; ... 
  
  Ah ! Parece que é uma PA de segunda ordem, a partir de um certo 
  ponto... 
  
  Vamos entender essa idéia. No longo prazo, o cavalo vai se afastando do 
  centro, e portanto ele pode cobrir uma área no máximo proporcional a N^2. 
  Isso por si só já justifica tentar achar uma PA de segunda ordem. O que é 
  interessante é que a parte perto do centro (depois do início, onde ainda há 
  um monte de buracos meio aleatórios) estará completamente coberta depois de 
  um certo tempo, e o que interessa é o que acontece nas coroas. Agora, tem 
  que justificar que as coroas têm uma espessura constante depois de passada 
  a 
  parte transiente 
  inicial. 
  
  Como eu usei um computador, e posso calcular mais do que n = 10 (por 
  exemplo 
  n = 100) e os 14 continuam até esse ponto. Para mim, isso é mais do que 
  suficiente para eu ter certeza que a resposta é essa, mas admito que falta 
  um argumento garantindo que basta observar um número finito de passos 
  para 
  acertar a recorrência. Eu diria que, como um cavalo completa a vizinhança 
  do ponto inicial (o 3x3 em volta da 
  origem) em uma quantidade finita de passos (basta chegar na profundidade 3 
  do grafo do Torres) a recorrência não pode ser de ordem muito maior do que 
  isso. Para melhorar, veja que a partir de 3 passos, o que temos é um 
  octógono, TODO preenchido, dos quadrados brancos (que são os únicos em que 
  o 
  cavalo pode estar!). Daí pra frente, não é difícil ver que a cada etapa 
  teremos um octógono com lado aumentando de 1 a cada vez. Veja também que a 
  partir do 3o termo da segunda diferença, só tem 14. Não é coincidência. 
  
  Agora, eu deixo a indução para você completar! 
  
  Abraços, 
  -- 
  Bernardo Freitas Paulo da Costa 
  
  -- 
  Esta mensagem foi verificada pelo sistema de antivírus e  acredita-se estar 
  livre de perigo. 
  
  
  = 
  Instruções para entrar na lista, sair da lista e usar a lista em 
  http://www.mat.puc-rio.br/~obmlistas/obm-l.html 
  = 
  
  
  --- 
  Este email está limpo de vírus e malwares porque a proteção do avast! 
  Antivírus está ativa. 
  http://www.avast.com 
  
  
  -- 
  Esta mensagem foi verificada pelo sistema de antivírus e 
   acredita-se estar livre de perigo. 
  
  
  = 
  Instruções para entrar na lista, sair da lista e usar a lista em 
  http://www.mat.puc-rio.br/~obmlistas/obm-l.html 
  = 
  
 
 -- 
 

[obm-l] Problema Legal

2012-05-17 Por tôpico Benedito Tadeu V. Freire
O problema abaixo apareceu na Lista de Problemas do pessoal da Argentina.

Problema 
Um dragão dá 100 moedas a um cavalheiro que ele mantém prisioneiro. A metade 
das moedas são mágicas, mas somente o dragão sabe quais são elas.
Cada dia, o cavalheiro tem que dividir as 100 moedas em duas pilhas, não 
necessariamente do mesmo tamanho.
Se algum dia as duas pilhas possuem o mesmo número de moedas mágicas ou as 
pilhas tem o mesmo número de moedas não mágicas, o cavalheiro ganha a liberdade.
Determinar se o cavalheiro pode ganhar sua liberdade em 50 dias ou menos.
E em 25 dias ou menos?

Benedito
-- 
Open WebMail Project (http://openwebmail.org)

 


[obm-l] Bom livro de Geometria sintética

2011-11-04 Por tôpico Benedito Tadeu V. Freire
Além dos livros já mencionados aqui, sugiro o livro:
Lesson in Geometry,  I. Plane Geometry - de Jacques Hadamard,  AMS 2008.
Pela  AMS (American Mathamatical Society), tem o livro correspondente das 
soluções dos problemas.
Maravilhoso!
Veja os dios livros citados no site da própria AMS: www.ams.org/bookstore. Ou 
ainda, leia os livros na íntegra, sem poder fazer cópia, no site 
www.googlebooks.com

Benedito

 


[obm-l] Problema legal! (Corrigindo o enunciado)

2011-02-28 Por tôpico Benedito Tadeu V. Freire
PROBLEMA

Cada uma das faces de uma folha de papel é dividida em três regiões limitadas 
por polígonos. Numa delas, uma das regiões limitada por um polígono é de cor 
branca, outra vermelha, e a terceirana outra verde.

Prove que, na outra face, é possível pintar uma das regiões polígonais de 
branco, outra de  vermelho, e a terceira de verde, de tal maneira que pelo 
menos um terço da área da folha de papel é colorido com a mesma cor em ambas as 
faces.

 


Re: [obm-l] Compra de livros

2010-08-12 Por tôpico Benedito Tadeu V. Freire
Henrique,

Tente: Livraria Cultura  ou Livraria da Física (os endereços você pode ver no 
google)
Benedito

-- 
Open WebMail Project (http://openwebmail.org)

-- Original Message ---
From: Henrique Rennó henrique.re...@gmail.com 
To: obm-l obm-l@mat.puc-rio.br 
Sent: Thu, 12 Aug 2010 09:15:21 -0300 
Subject: [obm-l] Compra de livros

 Gostaria de saber algum site onde posso encontrar diversos livros da 
 área de exatas, principalmente de matemática e realizar a compra 
 online. Estou à procura do livro Programação Linear de Manuel 
 Ramalhete. 
 
 -- 
 Henrique 
 
 = 
 Instruções para entrar na lista, sair da lista e usar a lista em 
 http://www.mat.puc-rio.br/~obmlistas/obm-l.html 
 = 
--- End of Original Message ---
 


Re: [obm-l] Re: [obm-l] Re: [obm-l] OFF-TOPIC - Raciocínio Lógico

2009-04-03 Por tôpico Benedito Tadeu V. Freire
Paulo César,

Os livros,com excessão de Book of Curious  Interesting Puzzles . David Wells. 
Dover. 1992.,  são livros de problemas,cada um deles com uma 
coleção interessantíssima. Acho que vale pena ver

Boa sorte

Benedito Freire

-- 
Open WebMail Project (http://openwebmail.org)

-- Original Message ---
From: Paulo Cesar pcesa...@gmail.com 
To: obm-l@mat.puc-rio.br 
Sent: Wed, 1 Apr 2009 22:21:55 -0300 
Subject: [obm-l] Re: [obm-l] Re: [obm-l] OFF-TOPIC - Raciocínio Lógico

 Muito obrigado, Benedito. 
   
 Você saberia dizer se os livros mencionados abordam teoricamente o tema? 
   
 Um abraço 
   
 PC 
 
   
 2009/3/31 benedito bened...@ufrnet.br


 
 Paulo César, 
   
 Veja alguns interessantes: 
 A) Em português: 
   
 1) A Dama e o Tigre e outros Problemas Lógicos,  de  Raymond Smullyan. 
 Jorge Zahar Editor. 1982 
 2) O Enígma de Sherazade, de  Raymond Smullyan. Jorge Zahar Editor. 
 3) Alice no País dos Enígmas,  de Raymond Smullyan. Jorge Zahar Editor. 
 4) Divertimentos Matemáticos, de Martin Gardner. IBRASA. 1967 
   
 Em inglês: 
 1) The Colossal Book of Short Puzzles - Martin Gardner. Norton.2006 
 2) Book of Curious  Interesting Puzzles . David Wells. Dover. 1992. 
   
 Ainda em inglês, o maravilhoso livro da Lógica Moderna: Sweet Reazon - A 
 field Guide to Modern Logic, de Tom Tymoczko  and  Jim Henle. Springer.2000. 
   
 Bom proveito. 
 Benedito 
 
 
 - Original Message - 
 From: Paulo Cesar 
 To: obm-l@mat.puc-rio.br 
 Sent: Tuesday, March 31, 2009 7:32 PM 
 Subject: [obm-l] OFF-TOPIC - Raciocínio Lógico 
 
 Olá mestres da lista 
   
 Gostaria de saber qual é o melhor livro de raciocínio lógico que posso 
 comprar. Estou a procura de um material mais aprofundado sobre o assunto. 
 O que vocês recomendam? 
   
 Um abraço pra todos 
   
 PC 
   
  
--- End of Original Message ---
 


Re: Livros

2001-03-19 Por tôpico Benedito Tadeu V. Freire



Alm dos livros j citados, veja, tambm, o
livro "Introduo  Teoria dos Nmeros" de
Jos Plnio de Oliveira Santos, da Coleo
Matemtica Universitria - IMPA.
Para comprar, entre em contato pelo endereo: www.impa.br
Benedito Freire
Igor Castro wrote:

Algum
sabe algum livro que tenha a teoria de congruencia, divisiblidade e etc,
e que seja bom para me indicar?




begin:vcard 
n:Freire;Benedito Tadeu
tel;fax:55 84 211 92 19
tel;work:55 84 215 38 20
x-mozilla-html:TRUE
org:Chefe do Departamento de Matemática;UFRN - Universidade Federal do Rio Grande do Norte
adr:;;Caixa Postal 1214;Natal;Rio Grande do Norte;59075-970;Brasil
version:2.1
email;internet:[EMAIL PROTECTED]
title:Benedito Tadeu Vasconcelos Freire
end:vcard



Nova Página

2001-02-19 Por tôpico Benedito Tadeu V. Freire


Se os colegas tiverem tempo, vejam a nova pgina da OLIMPADA DE MATEMTICA DO
RIO GRANDE DO NORTE: www.ufrn.br/olimpiada

Um fato altamente importante para ns  que  toda a pgina foi feita pelo
estudante do Curso de Matemtica da UFRN   Charles Csar Magno de Freitas.
Benedito Freire



josimat wrote:

 Para quem ainda nao estava aqui, no ultimo12/07, coloquei um problema bem
 parecido com esse, mas menos famoso. Houve apresentacao de algumas solucoes,
 duas delas brilhantes, uma do Carlos Vitor e outra de Eduardo Wagner. Quem
 quiser conferir, o problema eh este:

 Dado um tringulo ABC, com AB=AC. Tomam-se os pontos N e M pertencentes,
 respectivamente, aos lados AB e AC. Sendo a medida do ngulo BCN=30 graus,
 CBM=60 graus, NBM=20 graus. Determine a medida do ngulo BMN.
  []s JOSIMAR
 -Mensagem original-
 De: Exercicio [EMAIL PROTECTED]
 Para: [EMAIL PROTECTED] [EMAIL PROTECTED]
 Data: Quarta-feira, 14 de Fevereiro de 2001 01:55
 Assunto: Re: problema do tringulo.

 
 
  OKz... mas exatamente... em q lugar do lado AB eu vou traar o q vc
 indicou?
 Valeu!
 
 
 Falow's
 
   Exercicio 
 
  http://exercicio.cjb.net
  ICQ # 102856897
 
 
 
 
 
 
 
 Marcos Paulo escreveu:
 
  Trace BCR (com r pertencendo ao lado AB) e vc encontrara um monte de
  triangulos isosceles (inclusive um equilatero)... ai fica fcil...
  []'s MP
 
 
 


begin:vcard 
n:Freire;Benedito Tadeu
tel;fax:55 84 211 92 19
tel;work:55 84 215 38 20
x-mozilla-html:TRUE
org:Chefe do Departamento de Matemática;UFRN - Universidade Federal do Rio Grande do Norte
adr:;;Caixa Postal 1214;Natal;Rio Grande do Norte;59075-970;Brasil
version:2.1
email;internet:[EMAIL PROTECTED]
title:Benedito Tadeu Vasconcelos Freire
end:vcard



correção

2001-02-09 Por tôpico Benedito Tadeu V. Freire



Sem "usar congruncias".
Veja o seguinte: 3 elevado a 4  igual a
81, que deixa resto 1 quando dividido por 5. Portanto,
3 elevado a 4  da forma 5m + 1, com
m um inteiro. Por outro lado,
 3^59 = (3^4)^14 . 3^3 = (5m +1)^14. (5.5 +2).
Observe agora que, se voc elevar qualquer nmero
da forma (5m + 1) a uma potncia inteira positiva, resulta
num nmero da mesma forma (veja isso fcilmente usando o
Teorema do Binmio). Ou seja, (5m + 1) ^14 = 5k + 1, com k
inteiro.
Concluindo, 3^59 ser igual ao produto de dois nmeros:
(5k +1).(5.5 +2), que resulta num nmero da forma 5.n +2.
Portanto, o resto  dois.
Benedito Freire
filho wrote:

Problema-67 ( 
divertido resolver problemas )Qual o resto
de 3 ^ 59 na diviso por 5 ?No
livro a resposta  resto = 4.Encontrei
resto = 2 como resposta .Aguardo retorno, grato.




begin:vcard 
n:Freire;Benedito Tadeu
tel;fax:55 84 211 92 19
tel;work:55 84 215 38 20
x-mozilla-html:TRUE
org:Chefe do Departamento de Matemática;UFRN - Universidade Federal do Rio Grande do Norte
adr:;;Caixa Postal 1214;Natal;Rio Grande do Norte;59075-970;Brasil
version:2.1
email;internet:[EMAIL PROTECTED]
title:Benedito Tadeu Vasconcelos Freire
end:vcard



probleminha

2001-01-15 Por tôpico Benedito Tadeu V. Freire



Prezado Marcelo,
Pense no seguinte:
Qual  a paridade do nmero de mpares de 1
a 100?
Voc poderia encontrar como resultado final algum nmero
mpar?
Benedito
Marcelo Ferreira wrote:

Para
quem quiser pensar, segue o problema abaixo:
Escrevemos em um quadro negro os nmeros inteiros de 1 a 100. Depois
escolhemos dois nmeros a e
b escritos no quadro, apagamos a e b e
escrevemos a-b (agora h 99
inteiros escritos no quadro). Repetimos este
processo at que haja um nico
inteiro escrito no quadro. Prove que este
inteiro nunca pode ser igual a 1.




begin:vcard 
n:Freire;Benedito Tadeu
tel;fax:55 84 211 92 19
tel;work:55 84 215 38 20
x-mozilla-html:TRUE
org:Chefe do Departamento de Matemática;UFRN - Universidade Federal do Rio Grande do Norte
adr:;;Caixa Postal 1214;Natal;Rio Grande do Norte;59075-970;Brasil
version:2.1
email;internet:[EMAIL PROTECTED]
title:Benedito Tadeu Vasconcelos Freire
end:vcard



Re: [Fwd: Ajuda sobre teoria dos números...]

2000-05-05 Por tôpico Benedito Tadeu V. Freire



Caro Flvio,
Segue uma bibliografia que talvez seja interessante voc
acessar:
a) Introduo  Teoria dos Nmeros, de
Jos Plnio de Oliveira Santos -Rio de Janeiro - Coleo
Matemtica Universitria- SBM
b) An Introduction to the Theory of Numbers - Ivan M. Niven
and Herbert S. Zuckerman . New York. John Willey  Sons.
1991
Benedito Freire
Flavio Borges Botelho wrote:
Alexandre de Moura wrote:
Preciso de ajuda,
Podem me ajudar com a introduo e problemas sobre Teoria
dos nmeros . . . []
desde j muito obrigado,
Alexandre de Moura[EMAIL PROTECTED]







begin:vcard 
n:Freire;Benedito Tadeu
tel;fax:55 84 211 92 19
tel;work:55 84 215 38 20
x-mozilla-html:TRUE
org:Chefe do Departamento de Matemática;UFRN - Universidade Federal do Rio Grande do Norte
adr:;;Caixa Postal 1214;Natal;Rio Grande do Norte;59075-970;Brasil
version:2.1
email;internet:[EMAIL PROTECTED]
title:Benedito Tadeu Vasconcelos Freire
end:vcard



Re: Um Problema

2000-04-28 Por tôpico Benedito Tadeu V. Freire



Esqueci uma palavra fundamental: positiva!!

Problema
Para que valores de c a equao
8x + 5y = c admite uma nica soluo
inteira (i.e. x e y inteiros) positiva?
Benedito


begin:vcard 
n:Freire;Benedito Tadeu
tel;fax:55 84 211 92 19
tel;work:55 84 215 38 20
x-mozilla-html:TRUE
org:Chefe do Departamento de Matemática;UFRN - Universidade Federal do Rio Grande do Norte
adr:;;Caixa Postal 1214;Natal;Rio Grande do Norte;59075-970;Brasil
version:2.1
email;internet:[EMAIL PROTECTED]
title:Benedito Tadeu Vasconcelos Freire
end:vcard



Re: Um Problema

2000-04-28 Por tôpico Benedito Tadeu V. Freire

Prezado Prof. Barone,

Esqueci uma palavra fundamental: positiva. As soluções são positivas!!!:

Problema
Para que valores  de  c  a equação  8x + 5y = c admite uma única solução
inteira (i.e.  x  e  y inteiros)  positivas?

Obrigado pela observação.
Benedito Freire

Angelo Barone Netto wrote:

 Caro Benedito.
 Se admitir uma solucao inteira, admitira uma infinidade.

 Angelo Barone{\ --\ }NettoUniversidade de Sao Paulo
 Departamento de Matematica Aplicada   Instituto de Matematica e Estatistica
 Rua do Matao, 1010Butanta - Cidade Universitaria
 Caixa Postal 66 281   phone +55-11-818-6136
 05315-970 - Sao Paulo - SPfax +55-11-818-6131
 Agencia Cidade de Sao Paulo
 .


begin:vcard 
n:Freire;Benedito Tadeu
tel;fax:55 84 211 92 19
tel;work:55 84 215 38 20
x-mozilla-html:TRUE
org:Chefe do Departamento de Matemática;UFRN - Universidade Federal do Rio Grande do Norte
adr:;;Caixa Postal 1214;Natal;Rio Grande do Norte;59075-970;Brasil
version:2.1
email;internet:[EMAIL PROTECTED]
title:Benedito Tadeu Vasconcelos Freire
end:vcard



Um Problema

2000-04-27 Por tôpico Benedito Tadeu V. Freire


Problema
Para que valores  de  c  a equação  8x + 5y = c admite uma única solução
inteira (i.e.  x  e  y inteiros)?

Benedito


begin:vcard 
n:Freire;Benedito Tadeu
tel;fax:55 84 211 92 19
tel;work:55 84 215 38 20
x-mozilla-html:TRUE
org:Chefe do Departamento de Matemática;UFRN - Universidade Federal do Rio Grande do Norte
adr:;;Caixa Postal 1214;Natal;Rio Grande do Norte;59075-970;Brasil
version:2.1
email;internet:[EMAIL PROTECTED]
title:Benedito Tadeu Vasconcelos Freire
end:vcard



Re: [Fwd: Re: Problema: alterando levemente as hipóteses]

2000-04-19 Por tôpico Benedito Tadeu V. Freire

Prezado Flávio,

Apreciando seu esforço, tente mostrar usando números a partir de  m = 2000! Com um
pequeno arranjo você consegue os dois mil números desejados.

Benedito Freire

Flavio Borges Botelho wrote:

 2000 inteiros com a propriedade acima, teriam a forma:

 x == 1 (mod 3x5x7x9x11x13x15xx1999)

 Agora preciso provar que esse números vão ser primos entre si, ou gerar uma
 excessão desses números tal que sejam primos entre si.

 Já adianto que esse números claramente não vão ser primos entre si,
 porque vários seriam pares, mas talvez multiplicando o quociente por 2
 gerasse apenas números primos...

 Obrigado pela atenção,

 Flavio Botelho


begin:vcard 
n:Freire;Benedito Tadeu
tel;fax:55 84 211 92 19
tel;work:55 84 215 38 20
x-mozilla-html:TRUE
org:Chefe do Departamento de Matemática;UFRN - Universidade Federal do Rio Grande do Norte
adr:;;Caixa Postal 1214;Natal;Rio Grande do Norte;59075-970;Brasil
version:2.1
email;internet:[EMAIL PROTECTED]
title:Benedito Tadeu Vasconcelos Freire
end:vcard



Re: Problema: alterando levemente as hipóteses

2000-04-18 Por tôpico Benedito Tadeu V. Freire

Como garantir que a soma de um número qualquer deles seja um composto?
Exemplificando, se tomamos os primos  3, 13 e 7, então a soma 3 + 13 + 7  = 23
não é composto!
De fato, a modificação é sensível. A resposta, embora fácil,  não é tão direta.
Benedito Freire

Marcos Eike Tinen dos Santos wrote:

 Início da discussão:

 Observe que o único par que temos que é primo é o 2, sendo pertencente ao
 conjunto dos inteiros positivos.

 Então, podemos concluir de fato que todos os 2000 inteiros são ímpares, pois
 assim, me garantirá um número par, que neste caso será composto.

 Podemos, supor, então, como não há restrição, que esses 2000 inteiros são os
 próprios primos, já que mdc(p1,p2,p3,...,p2000) = 1

 Ats,
 Marcos Eike

 - Original Message -
 From: benedito [EMAIL PROTECTED]
 To: [EMAIL PROTECTED]
 Sent: Segunda-feira, 17 de Abril de 2000 20:21
 Subject: Problema: alterando levemente as hipóteses

  Alguns dias atrás enviei um problema que foi prontamente resolvido por um
  dos membros da lista. O que mostra que o pessoal está altamente ligado.
  Fazendo uma ligeira, mas sensível, modificação submeto-o aos membros da
 lista:
 
  Problema
  Encontre  2000  inteiros positivos relativamente primos, tais que todas as
  possíveis  somas de dois  ou mais desses números resultam em números
  compostos.
 
  Benedito Freire


begin:vcard 
n:Freire;Benedito Tadeu
tel;fax:55 84 211 92 19
tel;work:55 84 215 38 20
x-mozilla-html:TRUE
org:Chefe do Departamento de Matemática;UFRN - Universidade Federal do Rio Grande do Norte
adr:;;Caixa Postal 1214;Natal;Rio Grande do Norte;59075-970;Brasil
version:2.1
email;internet:[EMAIL PROTECTED]
title:Benedito Tadeu Vasconcelos Freire
end:vcard